Calculus Taylor Approximation Proof

Click For Summary
The discussion revolves around proving that the equation (1+x)^n = e^(n(x-(1/2)x^2)) holds true up to the x^2 order when |nx| < 1. Participants are using Taylor expansions for both sides, resulting in the same left-hand side expression of 1 + nx + (n/2)(n-1)x^2. The right-hand side also matches this expression, confirming the equality under the specified condition. Clarification is provided that the task is to demonstrate the equality given the condition, rather than proving the condition itself. The conversation ends with a request for hints on how to proceed with the proof.
Elvin Rivera
Messages
2
Reaction score
0
Member warned about deleting the template
1. The question is. Show that if |nx| <1, the following is exact up to (and including) the x^2 order. The hint giving says to use the Taylor Expansion for both sides of the equation2. (1+x)^n = e^n(x-(1/2)x^2) ; the n(x-(1/2)x^2) is all an exponent3. My first attempt was to take the taylor series of both sides. The left hand side is pretty easy.

1 + nx + (n/2)(n-1)x^2 = LHS

the right hand side gives the same thing though

f(0) = RHS = 1
f'(0) = n(1-x)e^n(x-(1/2)x^2) = n
f''(0) = -n*e^n(x-(1/2)x^2 + n^2 (1+x)^2 * e^n(x-(1/2)x^2) = -n + n^2

which in the taylor series gives the same as the LHS

1 + nx + (n/2)(n-1)x^2 = RHS

but now I'm stuck... how do I show that |nx| < 1
 
Last edited by a moderator:
Physics news on Phys.org
Elvin Rivera said:
1. The question is. Show that if |nx| <1, the following is exact up to (and including) the x^2 order. The hint giving says to use the Taylor Expansion for both sides of the equation2. (1+x)^n = e^n(x-(1/2)x^2) ; the n(x-(1/2)x^2) is all an exponent
Then use parentheses to make that clear: (1+ x)^n= e^(n(x- (1/2)x^2)).
3. My first attempt was to take the taylor series of both sides. The left hand side is pretty easy.

1 + nx + (n/2)(n-1)x^2 = LHS

the right hand side gives the same thing though

f(0) = RHS = 1
f'(0) = n(1-x)e^n(x-(1/2)x^2) = n
f''(0) = -n*e^n(x-(1/2)x^2 + n^2 (1+x)^2 * e^n(x-(1/2)x^2) = -n + n^2

which in the taylor series gives the same as the LHS

1 + nx + (n/2)(n-1)x^2 = RHS

but now I'm stuck... how do I show that |nx| < 1
You don't- that is not what is asked. You are asked to show that if |nx|< 1 then that equality is true, not the other way around.
 
Last edited by a moderator:
HallsofIvy said:
Then use parentheses to make that clear: (1+ x)^n= e^(n(x- (1/2)x^2)).
You don't- that is not what is asked. You are asked to show that if |nx|< 1 then that equality is true, not the other way around.

Any hint on how to start that?
 
Question: A clock's minute hand has length 4 and its hour hand has length 3. What is the distance between the tips at the moment when it is increasing most rapidly?(Putnam Exam Question) Answer: Making assumption that both the hands moves at constant angular velocities, the answer is ## \sqrt{7} .## But don't you think this assumption is somewhat doubtful and wrong?

Similar threads

Replies
2
Views
1K
  • · Replies 14 ·
Replies
14
Views
2K
Replies
1
Views
1K
Replies
6
Views
2K
Replies
2
Views
2K
  • · Replies 6 ·
Replies
6
Views
2K
Replies
5
Views
2K
  • · Replies 4 ·
Replies
4
Views
2K
  • · Replies 1 ·
Replies
1
Views
2K
  • · Replies 4 ·
Replies
4
Views
1K